disjoint cycles and equitable colorings in graphs
play

Disjoint Cycles and Equitable Colorings in Graphs H. Kierstead A. - PowerPoint PPT Presentation

Disjoint Cycles and Equitable Colorings in Graphs H. Kierstead A. Kostochka T. Molla M. Santana *E. Yeager University of British Columbia, Vancouver Canada Email: elyse@math.ubc.ca Japanese Conference on Combinatorics and its Applications


  1. Dirac-Erd˝ os Type Problems Kierstead-Kostochka-McConvey, 2016 (link) Let k ≥ 3 be an integer and G be a graph such that G does not contain two disjoint triangles. If V ≥ 2 k − V ≤ 2 k − 2 ≥ 2 k , then G contains k disjoint cycles. Question: do we really need to avoid disjoint triangles? Short answer: yes. Long answer: sometimes. Kierstead-Kostochka-McConvey, 2018 (link) Let k ≥ 2 be an integer and G be a graph with | G | ≥ 19 k and V ≥ 2 k − V ≤ 2 k − 2 ≥ 2 k . Then G contains k disjoint cycles. Open Characterize graphs G with V ≥ 2 k − V ≤ 2 k − 2 ≥ 2 k and no k disjoint cycles. 34 / 180

  2. Outline Disjoint Cycles 1 Corr´ adi-Hajnal Tolerance for some low-degree vertices Ore condition (minimum degree-sum of nonadjacent vertices) Generalized Degree-Sum Conditions Connectivity Neighborhood Union Chorded Cycles 2 Degree conditions Neighborhood Union Multiply Chorded Cycles Equitable Coloring 3 Definition Connection to Cycles 35 / 180

  3. Enomoto, Wang Corr´ adi-Hajnal, 1963 If G is a graph on n vertices with n ≥ 3 k and δ ( G ) ≥ 2 k , then G contains k disjoint cycles. 36 / 180

  4. Enomoto, Wang Corr´ adi-Hajnal, 1963 If G is a graph on n vertices with n ≥ 3 k and δ ( G ) ≥ 2 k , then G contains k disjoint cycles. σ 2 ( G ) := min { d ( x ) + d ( y ) : xy �∈ E ( G ) } 37 / 180

  5. Enomoto, Wang Corr´ adi-Hajnal, 1963 If G is a graph on n vertices with n ≥ 3 k and δ ( G ) ≥ 2 k , then G contains k disjoint cycles. σ 2 ( G ) := min { d ( x ) + d ( y ) : xy �∈ E ( G ) } Enomoto 1998, Wang 1999 If G is a graph on n vertices with n ≥ 3 k and σ 2 ( G ) ≥ 4 k − 1, then G contains k disjoint cycles. 38 / 180

  6. Enomoto, Wang Corr´ adi-Hajnal, 1963 If G is a graph on n vertices with n ≥ 3 k and δ ( G ) ≥ 2 k , then G contains k disjoint cycles. σ 2 ( G ) := min { d ( x ) + d ( y ) : xy �∈ E ( G ) } Enomoto 1998, Wang 1999 If G is a graph on n vertices with n ≥ 3 k and σ 2 ( G ) ≥ 4 k − 1, then G contains k disjoint cycles. Implies Corr´ adi-Hajnal 39 / 180

  7. Enomoto, Wang Corr´ adi-Hajnal, 1963 If G is a graph on n vertices with n ≥ 3 k and δ ( G ) ≥ 2 k , then G contains k disjoint cycles. σ 2 ( G ) := min { d ( x ) + d ( y ) : xy �∈ E ( G ) } Enomoto 1998, Wang 1999 If G is a graph on n vertices with n ≥ 3 k and σ 2 ( G ) ≥ 4 k − 1, then G contains k disjoint cycles. Implies Corr´ adi-Hajnal Low degree vertices OK as long as they’re in a clique 40 / 180

  8. Enomoto, Wang Corr´ adi-Hajnal, 1963 If G is a graph on n vertices with n ≥ 3 k and δ ( G ) ≥ 2 k , then G contains k disjoint cycles. σ 2 ( G ) := min { d ( x ) + d ( y ) : xy �∈ E ( G ) } Enomoto 1998, Wang 1999 If G is a graph on n vertices with n ≥ 3 k and σ 2 ( G ) ≥ 4 k − 1, then G contains k disjoint cycles. Implies Corr´ adi-Hajnal Low degree vertices OK as long as they’re in a clique With a little work, implies Dirac-Erd˝ os 41 / 180

  9. Enomoto, Wang Enomoto 1998, Wang 1999 If G is a graph on n vertices with n ≥ 3 k and σ 2 ( G ) ≥ 4 k − 1, then G contains k disjoint cycles. Proof (Enomoto) 42 / 180

  10. Enomoto, Wang Enomoto 1998, Wang 1999 If G is a graph on n vertices with n ≥ 3 k and σ 2 ( G ) ≥ 4 k − 1, then G contains k disjoint cycles. Proof (Enomoto) Edge-maximal counterexample 43 / 180

  11. Enomoto, Wang Enomoto 1998, Wang 1999 If G is a graph on n vertices with n ≥ 3 k and σ 2 ( G ) ≥ 4 k − 1, then G contains k disjoint cycles. Proof (Enomoto) Edge-maximal counterexample ◮ ( k − 1) disjoint cycles 44 / 180

  12. Enomoto, Wang Enomoto 1998, Wang 1999 If G is a graph on n vertices with n ≥ 3 k and σ 2 ( G ) ≥ 4 k − 1, then G contains k disjoint cycles. Proof (Enomoto) Edge-maximal counterexample ◮ ( k − 1) disjoint cycles ◮ Remaining graph at least 3 vertices 45 / 180

  13. Enomoto, Wang Enomoto 1998, Wang 1999 If G is a graph on n vertices with n ≥ 3 k and σ 2 ( G ) ≥ 4 k − 1, then G contains k disjoint cycles. Proof (Enomoto) Edge-maximal counterexample ◮ ( k − 1) disjoint cycles ◮ Remaining graph at least 3 vertices Minimize number of vertices in cycles 46 / 180

  14. Enomoto, Wang Enomoto 1998, Wang 1999 If G is a graph on n vertices with n ≥ 3 k and σ 2 ( G ) ≥ 4 k − 1, then G contains k disjoint cycles. Proof (Enomoto) Edge-maximal counterexample ◮ ( k − 1) disjoint cycles ◮ Remaining graph at least 3 vertices Minimize number of vertices in cycles Maximize longest path in remainder 47 / 180

  15. Enomoto, Wang Enomoto 1998, Wang 1999 If G is a graph on n vertices with n ≥ 3 k and σ 2 ( G ) ≥ 4 k − 1, then G contains k disjoint cycles. Sharpness: k k k 2 k − 1 48 / 180

  16. Enomoto, Wang Enomoto 1998, Wang 1999 If G is a graph on n vertices with n ≥ 3 k and σ 2 ( G ) ≥ 4 k − 1, then G contains k disjoint cycles. Sharpness: k k k 2 k − 1 3 k vertices 49 / 180

  17. Enomoto, Wang Enomoto 1998, Wang 1999 If G is a graph on n vertices with n ≥ 3 k and σ 2 ( G ) ≥ 4 k − 1, then G contains k disjoint cycles. Sharpness: k k k 2 k − 1 3 k vertices α ( G ) large 50 / 180

  18. Kierstead-Kostochka-Yeager, 2017 (link) Independence Number: Observation: α ( G ) ≥ n − 2 k + 1 ⇒ no k cycles 2 k − 1 51 / 180

  19. Kierstead-Kostochka-Yeager, 2017 (link) Independence Number: Observation: α ( G ) ≥ n − 2 k + 1 ⇒ no k cycles Enomoto 1998, Wang 1999 If G is a graph on n vertices with n ≥ 3 k and σ 2 ( G ) ≥ 4 k − 1, then G contains k disjoint cycles. 52 / 180

  20. Kierstead-Kostochka-Yeager, 2017 (link) Independence Number: Observation: α ( G ) ≥ n − 2 k + 1 ⇒ no k cycles Enomoto 1998, Wang 1999 If G is a graph on n vertices with n ≥ 3 k and σ 2 ( G ) ≥ 4 k − 1, then G contains k disjoint cycles. Kierstead-Kostochka-Yeager, 2017 (link) For k ≥ 4, if G is a graph on n vertices with n ≥ 3 k + 1 and σ 2 ( G ) ≥ 4 k − 3, then G contains k disjoint cycles if and only if α ( G ) ≤ n − 2 k . 53 / 180

  21. Kierstead-Kostochka-Yeager, 2017 Kierstead-Kostochka-Yeager, 2017 (link) For k ≥ 4, if G is a graph on n vertices with n ≥ 3 k + 1 and σ 2 ( G ) ≥ 4 k − 3, then G contains k disjoint cycles if and only if α ( G ) ≤ n − 2 k . 54 / 180

  22. Kierstead-Kostochka-Yeager, 2017 Kierstead-Kostochka-Yeager, 2017 (link) For k ≥ 4, if G is a graph on n vertices with n ≥ 3 k + 1 and σ 2 ( G ) ≥ 4 k − 3, then G contains k disjoint cycles if and only if α ( G ) ≤ n − 2 k . n ≥ 3 k + 1 k k k 2 k − 1 k 55 / 180

  23. Kierstead-Kostochka-Yeager, 2017 Kierstead-Kostochka-Yeager, 2017 (link) For k ≥ 4, if G is a graph on n vertices with n ≥ 3 k + 1 and σ 2 ( G ) ≥ 4 k − 3, then G contains k disjoint cycles if and only if α ( G ) ≤ n − 2 k . k = 1: 56 / 180

  24. Kierstead-Kostochka-Yeager, 2017 Kierstead-Kostochka-Yeager, 2017 (link) For k ≥ 4, if G is a graph on n vertices with n ≥ 3 k + 1 and σ 2 ( G ) ≥ 4 k − 3, then G contains k disjoint cycles if and only if α ( G ) ≤ n − 2 k . k = 2: u v 57 / 180

  25. Kierstead-Kostochka-Yeager, 2017 Kierstead-Kostochka-Yeager, 2017 (link) For k ≥ 4, if G is a graph on n vertices with n ≥ 3 k + 1 and σ 2 ( G ) ≥ 4 k − 3, then G contains k disjoint cycles if and only if α ( G ) ≤ n − 2 k . k = 3: 58 / 180

  26. Kierstead-Kostochka-Yeager, 2017 Kierstead-Kostochka-Yeager, 2017 (link) For k ≥ 4, if G is a graph on n vertices with n ≥ 3 k + 1 and σ 2 ( G ) ≥ 4 k − 3, then G contains k disjoint cycles if and only if α ( G ) ≤ n − 2 k . σ 2 = 4 k − 4: k − 3 2 r K 2 t k + 1 k + 3 2 r − 2 59 / 180

  27. Outline Disjoint Cycles 1 Corr´ adi-Hajnal Tolerance for some low-degree vertices Ore condition (minimum degree-sum of nonadjacent vertices) Generalized Degree-Sum Conditions Connectivity Neighborhood Union Chorded Cycles 2 Degree conditions Neighborhood Union Multiply Chorded Cycles Equitable Coloring 3 Definition Connection to Cycles 60 / 180

  28. Extending Enomoto-Wang Enomoto 1998, Wang 1999 If G is a graph on n vertices with n ≥ 3 k and σ 2 ( G ) ≥ 4 k − 1, then G contains k disjoint cycles. σ 2 ( G ) := min { d ( x ) + d ( y ) : xy �∈ E ( G ) } 61 / 180

  29. Extending Enomoto-Wang Enomoto 1998, Wang 1999 If G is a graph on n vertices with n ≥ 3 k and σ 2 ( G ) ≥ 4 k − 1, then G contains k disjoint cycles. σ 2 ( G ) := min { d ( x ) + d ( y ) : xy �∈ E ( G ) } � � � σ t ( G ) = min d ( V ) : I is an independent set of size t v ∈ I 62 / 180

  30. Extending Enomoto-Wang Enomoto 1998, Wang 1999 If G is a graph on n vertices with n ≥ 3 k and σ 2 ( G ) ≥ 4 k − 1, then G contains k disjoint cycles. σ 2 ( G ) := min { d ( x ) + d ( y ) : xy �∈ E ( G ) } � � � σ t ( G ) = min d ( V ) : I is an independent set of size t v ∈ I Conjecture: Gould, Hirohata, Keller 2018 (link) Let G be a graph of sufficiently large order. If σ t ( G ) ≥ 2 kt − t + 1 for any two integers k ≥ 2 and t ≥ 1, then G contains k disjoint cycles. 63 / 180

  31. Extending Enomoto-Wang Enomoto 1998, Wang 1999 If G is a graph on n vertices with n ≥ 3 k and σ 2 ( G ) ≥ 4 k − 1, then G contains k disjoint cycles. σ 2 ( G ) := min { d ( x ) + d ( y ) : xy �∈ E ( G ) } � � � σ t ( G ) = min d ( V ) : I is an independent set of size t v ∈ I Conjecture: Gould, Hirohata, Keller 2018 (link) Let G be a graph of sufficiently large order. If σ t ( G ) ≥ 2 kt − t + 1 for any two integers k ≥ 2 and t ≥ 1, then G contains k disjoint cycles. t = 1: Corr´ adi-Hajnal t = 2: Enomoto-Wang 64 / 180

  32. Extending Enomoto-Wang Enomoto 1998, Wang 1999 If G is a graph on n vertices with n ≥ 3 k and σ 2 ( G ) ≥ 4 k − 1, then G contains k disjoint cycles. σ 2 ( G ) := min { d ( x ) + d ( y ) : xy �∈ E ( G ) } � � � σ t ( G ) = min d ( V ) : I is an independent set of size t v ∈ I Conjecture: Gould, Hirohata, Keller 2018 (link) Let G be a graph of sufficiently large order. If σ t ( G ) ≥ 2 kt − t + 1 for any two integers k ≥ 2 and t ≥ 1, then G contains k disjoint cycles. t = 1: Corr´ adi-Hajnal t = 2: Enomoto-Wang t = 3: Fujita, Matsumura, Tsugaki, Yamashita 2006 (link) t = 4: proved in paper as evidence for conjecture 65 / 180

  33. Ma, Yan Conjecture: Gould, Hirohata, Keller 2018 (link) Let G be a graph of sufficiently large order. If σ t ( G ) ≥ 2 kt − t + 1 for any two integers k ≥ 2 and t ≥ 1, then G contains k disjoint cycles. True for t ≤ 4. 66 / 180

  34. Ma, Yan Conjecture: Gould, Hirohata, Keller 2018 (link) Let G be a graph of sufficiently large order. If σ t ( G ) ≥ 2 kt − t + 1 for any two integers k ≥ 2 and t ≥ 1, then G contains k disjoint cycles. True for t ≤ 4. Ma, Yan 2018+ (link) Let G be a graph with | G | ≥ (2 t + 1) k . If σ t ( G ) ≥ 2 kt − t + 1 for any two integers k ≥ 2 and t ≥ 5, then G contains k disjoint cycles. 67 / 180

  35. Ma, Yan Conjecture: Gould, Hirohata, Keller 2018 (link) Let G be a graph of sufficiently large order. If σ t ( G ) ≥ 2 kt − t + 1 for any two integers k ≥ 2 and t ≥ 1, then G contains k disjoint cycles. True for t ≤ 4. Ma, Yan 2018+ (link) Let G be a graph with | G | ≥ (2 t + 1) k . If σ t ( G ) ≥ 2 kt − t + 1 for any two integers k ≥ 2 and t ≥ 5, then G contains k disjoint cycles. Proof In an edge-maximal counterexample, choose k − 1 disjoint cycles such that number of vertices in cycles is minimal, and number of connected components in remaining graph is minimal 68 / 180

  36. Ma, Yan Conjecture: Gould, Hirohata, Keller 2018 (link) Let G be a graph of sufficiently large order. If σ t ( G ) ≥ 2 kt − t + 1 for any two integers k ≥ 2 and t ≥ 1, then G contains k disjoint cycles. True for t ≤ 4. Ma, Yan 2018+ (link) Let G be a graph with | G | ≥ (2 t + 1) k . If σ t ( G ) ≥ 2 kt − t + 1 for any two integers k ≥ 2 and t ≥ 5, then G contains k disjoint cycles. Degree-sum condition is sharp: 69 / 180

  37. Ma, Yan Conjecture: Gould, Hirohata, Keller 2018 (link) Let G be a graph of sufficiently large order. If σ t ( G ) ≥ 2 kt − t + 1 for any two integers k ≥ 2 and t ≥ 1, then G contains k disjoint cycles. True for t ≤ 4. Ma, Yan 2018+ (link) Let G be a graph with | G | ≥ (2 t + 1) k . If σ t ( G ) ≥ 2 kt − t + 1 for any two integers k ≥ 2 and t ≥ 5, then G contains k disjoint cycles. Degree-sum condition is sharp: 2 k − 1 70 / 180

  38. Ma, Yan Conjecture: Gould, Hirohata, Keller 2018 (link) Let G be a graph of sufficiently large order. If σ t ( G ) ≥ 2 kt − t + 1 for any two integers k ≥ 2 and t ≥ 1, then G contains k disjoint cycles. True for t ≤ 4. Ma, Yan 2018+ (link) Let G be a graph with | G | ≥ (2 t + 1) k . If σ t ( G ) ≥ 2 kt − t + 1 for any two integers k ≥ 2 and t ≥ 5, then G contains k disjoint cycles. Degree-sum condition is sharp: 2 k − 1 71 / 180

  39. Ma, Yan Conjecture: Gould, Hirohata, Keller 2018 (link) Let G be a graph of sufficiently large order. If σ t ( G ) ≥ 2 kt − t + 1 for any two integers k ≥ 2 and t ≥ 1, then G contains k disjoint cycles. True for t ≤ 4. Ma, Yan 2018+ (link) Let G be a graph with | G | ≥ (2 t + 1) k . If σ t ( G ) ≥ 2 kt − t + 1 for any two integers k ≥ 2 and t ≥ 5, then G contains k disjoint cycles. Open What is the best possible bound on | G | in the Ma-Yan Theorem? Can we characterize graphs G with σ t ( G ) ≥ 2 kt − t + 1 but no k disjoint cycles? 72 / 180

  40. Outline Disjoint Cycles 1 Corr´ adi-Hajnal Tolerance for some low-degree vertices Ore condition (minimum degree-sum of nonadjacent vertices) Generalized Degree-Sum Conditions Connectivity Neighborhood Union Chorded Cycles 2 Degree conditions Neighborhood Union Multiply Chorded Cycles Equitable Coloring 3 Definition Connection to Cycles 73 / 180

  41. Dirac: (2 k − 1)-connected without k disjoint cycles Dirac, 1963 (link) What (2 k − 1)-connected graphs do not have k disjoint cycles? 74 / 180

  42. Dirac: (2 k − 1)-connected without k disjoint cycles Dirac, 1963 (link) What (2 k − 1)-connected graphs do not have k disjoint cycles? Observation: G is (2 k − 1) connected 75 / 180

  43. Dirac: (2 k − 1)-connected without k disjoint cycles Dirac, 1963 (link) What (2 k − 1)-connected graphs do not have k disjoint cycles? Observation: G is (2 k − 1) connected ⇒ δ ( G ) ≥ 2 k − 1 76 / 180

  44. Dirac: (2 k − 1)-connected without k disjoint cycles Dirac, 1963 (link) What (2 k − 1)-connected graphs do not have k disjoint cycles? Observation: G is (2 k − 1) connected ⇒ δ ( G ) ≥ 2 k − 1 ⇒ σ 2 ( G ) ≥ 4 k − 2 77 / 180

  45. Dirac: (2 k − 1)-connected without k disjoint cycles Dirac, 1963 (link) What (2 k − 1)-connected graphs do not have k disjoint cycles? Observation: G is (2 k − 1) connected ⇒ δ ( G ) ≥ 2 k − 1 ⇒ σ 2 ( G ) ≥ 4 k − 2 KKY: Holds for σ 2 ( G ) ≥ 4 k − 3 78 / 180

  46. Dirac: (2 k − 1)-connected without k disjoint cycles Dirac, 1963 (link) What (2 k − 1)-connected graphs do not have k disjoint cycles? Answer to Dirac’s Question for Simple Graphs (KKY 2017) Let k ≥ 2. Every graph G with ( i ) | G | ≥ 3 k and ( ii ) δ ( G ) ≥ 2 k − 1 contains k disjoint cycles if and only if if k is odd and | G | = 3 k , then G � = 2 K k ∨ K k , and α ( G ) ≤ | G | − 2 k , and if k = 2 then G is not a wheel. k k k 2 k − 1 79 / 180

  47. Dirac: (2 k − 1)-connected without k disjoint cycles Dirac, 1963 (link) What (2 k − 1)-connected graphs do not have k disjoint cycles? Answer to Dirac’s Question for Simple Graphs (KKY 2017) Let k ≥ 2. Every graph G with ( i ) | G | ≥ 3 k and ( ii ) δ ( G ) ≥ 2 k − 1 contains k disjoint cycles if and only if if k is odd and | G | = 3 k , then G � = 2 K k ∨ K k , and α ( G ) ≤ | G | − 2 k , and if k = 2 then G is not a wheel. Further: characterization for multigraphs 80 / 180

  48. Simple Graphs → Multigraphs Idea: Take all 1-vertex cycles 81 / 180

  49. Simple Graphs → Multigraphs Idea: Take all 1-vertex cycles 82 / 180

  50. Simple Graphs → Multigraphs Idea: Take all 1-vertex cycles Take as many 2-vertex cycles as possible (maximum matching) 83 / 180

  51. Simple Graphs → Multigraphs Idea: Take all 1-vertex cycles Take as many 2-vertex cycles as possible (maximum matching) 84 / 180

  52. Simple Graphs → Multigraphs Idea: Take all 1-vertex cycles Take as many 2-vertex cycles as possible (maximum matching) What’s left is a simple graph 85 / 180

  53. (2 k − 1)-connected multigraphs with no k disjoint cycles Answer to Dirac’s Question for multigraphs: Kierstead-Kostochka-Yeager 2015 (link) Let k ≥ 2 and n ≥ k . Let G be an n -vertex graph with simple degree at least 2 k − 1 and no loops. Let F be the simple graph induced by the strong edgs of G , α ′ = α ′ ( F ), and k ′ = k − α ′ . Then G does not contain k disjoint cycles if and only if one of the following holds: n + α ′ < 3 k ; | F | = 2 α ′ (i.e., F has a perfect matching) and either (i) k ′ is odd and G − F = Y k ′ , k ′ , or (ii) k ′ = 2 < k and G − F is a wheel with 5 spokes; G is extremal and either (i) some big set is not incident to any strong edge, or (ii) for some two distinct big sets I j and I j ′ , all strong edges intersecting I j ∪ I j ′ have a common vertex outside of I j ∪ I j ′ ; n = 2 α ′ + 3 k ′ , k ′ is odd, and F has a superstar S = { v 0 , . . . , v s } with center v 0 such that either (i) G − ( F − S + v 0 ) = Y k ′ +1 , k ′ , or (ii) s = 2, v 1 v 2 ∈ E ( G ), G − F = Y k ′ − 1 , k ′ and G has no edges between { v 1 , v 2 } and the set X 0 in G − F ; k = 2 and G is a wheel, where some spokes could be strong edges; k ′ = 2, | F | = 2 α ′ + 1 = n − 5, and G − F = C 5 . 86 / 180

  54. k ′ odd, F has a perfect matching Example: k = 8, α ′ = 3, k ′ = 5. k ′ k ′ k ′ 87 / 180

  55. Big independent set, incident to no multiple edges 2 k − 1 88 / 180

  56. Wheel, with possibly some spokes multiple Example: k = 2 89 / 180

  57. Dirac: (2 k − 1)-connected without k disjoint cycles Dirac, 1963 (link) What (2 k − 1)-connected multigraphs do not have k disjoint cycles? Kierstead-Kostochka-Yeager 2015 (link) Characterization of multigraphs without k disjoint cycles that have minimum simple degree at least 2 k − 1. That is, the underlying simple graph G has δ ( G ) ≥ 2 k − 1. 90 / 180

  58. Dirac: (2 k − 1)-connected without k disjoint cycles Dirac, 1963 (link) What (2 k − 1)-connected multigraphs do not have k disjoint cycles? Kierstead-Kostochka-Yeager 2015 (link) Characterization of multigraphs without k disjoint cycles that have minimum simple degree at least 2 k − 1. That is, the underlying simple graph G has δ ( G ) ≥ 2 k − 1. Kierstead-Kostochka-Molla-Yager 2018+ (link) Characterization of multigraphs without k disjoint cycles that have minimum simple degree sum of nonadjacent vertices at least 4 k − 3. That is, the underlying simple graph G has σ 2 ( G ) ≥ 4 k − 3. 91 / 180

  59. Dirac: (2 k − 1)-connected without k disjoint cycles Dirac, 1963 (link) What (2 k − 1)-connected multigraphs do not have k disjoint cycles? Kierstead-Kostochka-Yeager 2015 (link) Characterization of multigraphs without k disjoint cycles that have minimum simple degree at least 2 k − 1. That is, the underlying simple graph G has δ ( G ) ≥ 2 k − 1. Kierstead-Kostochka-Molla-Yager 2018+ (link) Characterization of multigraphs without k disjoint cycles that have minimum simple degree sum of nonadjacent vertices at least 4 k − 3. That is, the underlying simple graph G has σ 2 ( G ) ≥ 4 k − 3. Open Do the other results in this talk generalize nicely to multigraphs? 92 / 180

  60. Outline Disjoint Cycles 1 Corr´ adi-Hajnal Tolerance for some low-degree vertices Ore condition (minimum degree-sum of nonadjacent vertices) Generalized Degree-Sum Conditions Connectivity Neighborhood Union Chorded Cycles 2 Degree conditions Neighborhood Union Multiply Chorded Cycles Equitable Coloring 3 Definition Connection to Cycles 93 / 180

  61. Neighborhood Union Faudree-Gould, 2005 (link) If G has n ≥ 3 k vertices and | N ( x ) ∪ N ( y ) | ≥ 3 k for all nonadjacent pairs of vertices x , y , then G contains k disjoint cycles. 94 / 180

  62. Neighborhood Union Faudree-Gould, 2005 (link) If G has n ≥ 3 k vertices and | N ( x ) ∪ N ( y ) | ≥ 3 k for all nonadjacent pairs of vertices x , y , then G contains k disjoint cycles. x y 95 / 180

  63. Neighborhood Union Faudree-Gould, 2005 (link) If G has n ≥ 3 k vertices and | N ( x ) ∪ N ( y ) | ≥ 3 k for all nonadjacent pairs of vertices x , y , then G contains k disjoint cycles. x y d ( x ) + d ( y ) = 6 96 / 180

  64. Neighborhood Union Faudree-Gould, 2005 (link) If G has n ≥ 3 k vertices and | N ( x ) ∪ N ( y ) | ≥ 3 k for all nonadjacent pairs of vertices x , y , then G contains k disjoint cycles. x y d ( x ) + d ( y ) = 6 | N ( x ) ∪ N ( y ) | = 4 97 / 180

  65. Neighborhood Union Faudree-Gould, 2005 (link) If G has n ≥ 3 k vertices and | N ( x ) ∪ N ( y ) | ≥ 3 k for all nonadjacent pairs of vertices x , y , then G contains k disjoint cycles. Neither stronger nor weaker than Corr´ adi-Hajnal. If δ ( G ) = 2 k , then xy �∈ E ( G ) {| N ( x ) ∪ N ( y ) |} ≥ 2 k . min If | N ( x ) ∪ N ( y ) | ≥ 3 k , then δ ( G ) ≥ 0. 98 / 180

  66. Neighborhood Union Faudree-Gould, 2005 (link) If G has n ≥ 3 k vertices and | N ( x ) ∪ N ( y ) | ≥ 3 k for all nonadjacent pairs of vertices x , y , then G contains k disjoint cycles. Proof In an edge-maximal counterexample, choose k − 1 disjoint cycles such that number of vertices in cycles is minimal, and number of connected components in remaining graph is minimal 99 / 180

  67. Neighborhood Union Faudree-Gould, 2005 (link) If G has n ≥ 3 k vertices and | N ( x ) ∪ N ( y ) | ≥ 3 k for all nonadjacent pairs of vertices x , y , then G contains k disjoint cycles. Sharpness: K 3 k − 4 K 5 100 / 180

Download Presentation
Download Policy: The content available on the website is offered to you 'AS IS' for your personal information and use only. It cannot be commercialized, licensed, or distributed on other websites without prior consent from the author. To download a presentation, simply click this link. If you encounter any difficulties during the download process, it's possible that the publisher has removed the file from their server.

Recommend


More recommend